Civil Code III Flashcards

You may prefer our related Brainscape-certified flashcards:
1
Q

A synallagmatic contract is a contract

A. under which the parties obligate themselves reciprocally.
B. that is entirely oral.
C. that is given no special designation, such as a designation as a sale, lease, loan or insurance.
D. that contains an obligation subject to a condition that depends solely on the whim of the obligor.
E. that is made by a party without contractual capacity.

A

A. under which the parties obligate themselves reciprocally.

The Louisiana Civil Code uses the phrase “synallagmatic contract” to describe a lease. It is a bilateral contract. Each party is bound to render a performance. Response C describes an innominate contract and D describes a contract with a potestative condition. Response B is incorrect because whether a contract is written or oral has no bearing on its classification as synallagmatic.

How well did you know this?
1
Not at all
2
3
4
5
Perfectly
2
Q

Lesion may be claimed

A. in the case of the sale of any immovable, unless the seller has renounced the right to claim it.
B. only in the case of the sale of a corporeal immovable.
C. within the prescriptive period of four years from the date of the sale.
D. by the seller of corporeal property that is movable or immovable.
E. by the seller of immovable property that is corporeal or incorporeal.

A

B. only in the case of the sale of a corporeal immovable.

This question calls for simple recall of the prescription that applies to a lesion claim and the circumstances under which such a claim may be brought.

How well did you know this?
1
Not at all
2
3
4
5
Perfectly
3
Q

Under the Louisiana Civil Code, a lessee may assign its right under a lease

A. only if the lease expressly permits assignment.
B. unless the lease prohibits assignment.
C. regardless of what the lease may provide with respect to assignment.
D. with the consent of the lessor, not to be unreasonably withheld.
E. with the consent of the lessor, not to be unreasonably withheld, except that the lease may provide a contrary rule.

A

B. unless the lease prohibits assignment.

Both assignment and sublease are permitted in Louisiana, unless the lease provides otherwise. D and E may seem like attractive options at first blush. The doctrine of abuse of right prevents a lessor from unreasonably withholding his consent to a sublease or assignment. But this doctrine does not kick in unless the lease provides that an assignment or sublease may be perfected only upon the lessor’s consent.

How well did you know this?
1
Not at all
2
3
4
5
Perfectly
4
Q

Under the Louisiana Uniform Commercial Code —Secured Transactions, the general rule of priority is that conflicting perfected security interests rank

A. according to priority in time of filing or perfection.
B. according to priority in time of filing.
C. according to priority in time of perfection.
D. according to priority in time of achieving both filing and perfection.
E. according to priority in time of achieving both attachment and perfection.

A

A. according to priority in time of filing or perfection.

The rule is “first to file or perfect” wins, as it is possible (and sometimes required) to perfect other than by filing, and it is possible to file before attachment and technical perfection and yet beat an intervening perfected creditor.

How well did you know this?
1
Not at all
2
3
4
5
Perfectly
5
Q

Which of the following is essential to the establishment of a conventional mortgage between the parties?

A. a statement that the promissory note(s) secured by the mortgage has (have) been paraphed for identification with the mortgage.
B. the signatures of both mortgagor and mortgagee.
C. filing of the mortgage in the mortgage records of the parish in which the property is located.
D. execution of the mortgage in authentic form.
E. None of the above.

A

E. None of the above.

Paraphs are no longer necessary, only the mortgagor must sign, filing the mortgage is for effectiveness against third parties, not the original parties, and an act under private signature is fully sufficient form for a mortgage.

How well did you know this?
1
Not at all
2
3
4
5
Perfectly
6
Q

A mandate necessarily terminates upon

A. death of the principal.
B. incapacity of the principal.
C. the filing of a revocatory action against the principal.
D. any condition that makes express revocation of the mandate impossible or impractical.
E. the expiration of ten years from the date the mandate was granted.

A

A. death of the principal.

Mandate generally terminates upon the death of the principal. This answer is not perfect, as the mandatary must still complete undertakings he began before the principal’s death if delay would injure the principal’s interest. But it is the “best” answer here. Note that response B is incorrect because not all incapacities of the principal terminate the mandate; interdiction does, but there may be other causes of the principal’s incapacity.

How well did you know this?
1
Not at all
2
3
4
5
Perfectly
7
Q

An offer of reward made to the public is binding upon the offeror

A. only if the person who performs the requested act knows of the offer.
B. even if the person who performs the requested act does not know of the offer.
C. only if the person who performs the requested act relies to his detriment on the offer in performing the requested act.
D. even if the offeror, before completion of the requested act, revokes the offer by the same or an equally effective means as the offer.
E. only if the formal requirements of a donation inter vivos have been satisfied in the making of the offer.

A

B. even if the person who performs the requested act does not know of the offer.

This question calls for simple recall of the general rule that offers of reward made to the public are binding regardless of the knowledge of the party who renders the requested performance.

How well did you know this?
1
Not at all
2
3
4
5
Perfectly
8
Q

Which of the following is a correct statement of the law?

A. The action in redhibition against a bad faith seller prescribes in four years from the day
delivery of the thing sold was made to the buyer or one year from the day the defect was discovered by the buyer, whichever occurs first.
B. In an action for recission because of a redhibitory defect, the court may limit the remedy of the buyer to reduction of the purchase price.
C. The seller is presumed to have known of a redhibitory defect if it appears within three days from the time of delivery of the thing sold.
D. An exclusion or limitation of the warranty against redhibitory vices is not permitted in consumer transactions.
E. All of the above.

A

B. In an action for recission because of a redhibitory defect, the court may limit the remedy of the buyer to reduction of the purchase price.

A buyer who seeks reduction of the purchase price for redhibition may not later seek dissolution, but a buyer who seeks dissolution may be compelled to accept a price reduction. Response A describes the prescriptive period that applies to a good faith seller, not a bad faith seller. Response C is rather tricky. The rule is that a redhibitory defect is presumed to have existed at the time of delivery if it appears within three days of delivery. The presumption has nothing to do with the knowledge of the seller.

How well did you know this?
1
Not at all
2
3
4
5
Perfectly
9
Q

Which of the following is not a correct statement of the law?

A. Stipulated damages must be reduced by the court if the obligor proves that the amount of the stipulated damages exceeds the obligee’s actual damages.
B. Stipulated damages for non-performance may be reduced in proportion to the benefit derived by the obligee from any partial performance rendered by the obligor.
C. Nullity of the principal obligation renders a clause for stipulated damages null.
D. An obligee may not avail himself of a clause stipulating damages for delay unless the obligor has been put in default.
E. An obligor whose failure to perform the principal obligation is justified by a valid excuse is also relieved of liability for stipulated damages.

A

A. Stipulated damages must be reduced by the court if the obligor proves that the amount of the stipulated damages exceeds the obligee’s actual damages.

Stipulated damages will be acceptable even if they do not precisely match the obligee’s actual damages. As long as they are not so unreasonable as to be violative of public policy, they are enforceable.

How well did you know this?
1
Not at all
2
3
4
5
Perfectly
10
Q

Which of the following is an example of circumstances under which a natural obligation arises?

A. A contract to sell immovable property is unenforceable because it is not in writing.
B. A borrower enters into an unenforceable agreement to pay interest at a usurious rate.
C. A civil obligation has been extinguished by prescription.
D. A contract is unenforceable because it is predicated upon unlawful cause.
E. All of the above.

A

C. A civil obligation has been extinguished by prescription.

A natural obligation exists only where “the law implies” a moral duty to render performance. Obligations extinguished by prescription are the most common examples of natural obligations. The law certainly does not imply a moral duty to render performance on an unlawful cause. The illegality of the cause provides good reason for refusing to give the agreement any effect.

How well did you know this?
1
Not at all
2
3
4
5
Perfectly
11
Q

A natural obligation:

A. arises from circumstances in which the law implies a particular moral duty to render a performance.
B. is enforceable by judicial action.
C. arises, for example, when a party agrees to a performance that is contra bonos mores.
D. is not onerous cause for a promise to fulfill it.
E. has all of the characteristics and effects described above.

A

A. arises from circumstances in which the law implies a particular moral duty to render a performance.

Choice A uses the exact codal definition of a natural obligation. C is incorrect because a natural obligation exists only when the law implies a moral duty, not when a party agrees to an immoral performance (a performance contra bonos mores). B and D are incorrect because, in contrast with civil obligations, natural obligations are not judicially enforceable, yet promises to fulfill them will amount to onerous cause.

How well did you know this?
1
Not at all
2
3
4
5
Perfectly
12
Q

A resolutory condition:

A. prevents the enforcement of an obligation until an uncertain event occurs.
B. causes an obligation to come to an end when an uncertain event occurs.
C. that depends solely on the whim of the obligor makes the obligation null.
D. is implied in every synallagmatic contract.
E. is satisfied when a corporate officer produces authentic evidence of a resolution authorizing his execution of a contract.

A

B. causes an obligation to come to an end when an uncertain event occurs.

Choice A describes a suspensive condition, and choice C describes the rule of nullity for suspensive conditions only. D is incorrect, as synallagmatic (in essence, bilateral) contracts need not be conditional ones.

How well did you know this?
1
Not at all
2
3
4
5
Perfectly
13
Q

Remission of a debt:

A. granted to a surety releases the principal obligor and all other sureties.
B. granted to one surety releases the other sureties only to the extent of the contribution the other sureties might have recovered from the surety to whom the remission was granted.
C. is effective only when express.
D. is not effective until the obligor has expressly accepted the remission.
E. is presumed to arise from the release of a real security given for performance of the debt.

A

B. granted to one surety releases the other sureties only to the extent of the contribution the other sureties might have recovered from the surety to whom the remission was granted.

The other responses are clearly wrong. C is incorrect because a remission may be tacit. D is incorrect because acceptance is presumed. Note that E is not entirely accurate, as releasing real security releases a commercial surety only to the extent that the release actually injures the surety, and the “performance of the debt” might have been partial.

How well did you know this?
1
Not at all
2
3
4
5
Perfectly
14
Q

Which of the following is a correct statement of the law?

A. Novation is the extinguishment of an existing obligation by the substitution of a new one.
B. Objective novation takes place when a new obligor is substituted for a prior obligor who is discharged by the obligee.
C. A novation made by the obligee and one of the obligors of a solidary obligation does not effect a release of the other solidary obligors.
D. Execution of a new writing and modification of an obligation, made without intention to extinguish it, are examples of novation.
E. All of the above.

A

A. Novation is the extinguishment of an existing obligation by the substitution of a new one.

Choice A memorializes the codal definition of a novation. B describes a subjective novation. C is incorrect, because novation of a solidary obligation releases all solidary obligors. D describes “mere modifications,” as distinguished from a novation.

How well did you know this?
1
Not at all
2
3
4
5
Perfectly
15
Q

Compensation:

A. between an obligee and a surety extinguishes the obligation of the principal obligor.
B. takes place only when two obligations are to be performed at the same place and arise from the same source.
C. can neither take place nor be renounced to the prejudice of rights previously acquired by third parties.
D. causes an obligation to be extinguished when the qualities of obligor and obligee of the obligation are united in the same person.
E. has all of the attributes and effects described above.

A

C. can neither take place nor be renounced to the prejudice of rights previously acquired by third parties.

A inappropriately flips the principle that extinction of the principal obligation extinguishes its accessories; the reverse is not true. D describes confusion, not compensation. B is incorrect because compensation may operate regardless of differing sources or places of performance.

How well did you know this?
1
Not at all
2
3
4
5
Perfectly
16
Q

In the case of the sale of an immovable:

A. the seller may disclaim the warranty against eviction but may not absolve himself of the obligation to return the purchase price if eviction occurs.
B. the seller may absolve himself of responsibility for an eviction that is occasioned by his own act, provided that he does so by clear and unambiguous language that is brought to the attention of the buyer and explained to him.
C. a buyer who avails himself of the warranty against eviction may recover from the seller the price he paid, the value of any fruits he had to return to the third person who evicted him, and also other damages sustained because of the eviction with the exception of any increase in value of the thing lost.
D. the liability of the seller upon the buyer’s eviction is limited to a return of double the purchase price.
E. the buyer’s action to enforce against a good faith seller the warranty eviction prescribes one year from the date of the sale.

A

C. a buyer who avails himself of the warranty against eviction may recover from the seller the price he paid, the value of any fruits he had to return to the third person who evicted him, and also other damages sustained because of the eviction with the exception of any increase in value of the thing lost.

Although it could be more precise, C is the best choice here. A buyer who successfully demonstrates breach of this warranty does receive the sums detailed, but damages only if he was unaware of the danger of eviction. A is incorrect because a sale at the buyer’s peril and risk even relieves the seller of the responsibility of returning the price. B is incorrect because a seller may not escape responsibility for his own act even through waivers of warranty. E describes the prescriptive period for breach of the warranty against redhibitory defects.

How well did you know this?
1
Not at all
2
3
4
5
Perfectly
17
Q

Which of the following is a correct statement of the law?

A. There is a management of affairs when a person acts with tacit authority to protect the interests of another, in the reasonable belief that the latter has authorized the action.
B. Management of affairs is synonymous with unjust enrichment.
C. A management of affairs is subject to the rules of mandate to the extent that those rules are compatible with management of affairs.
D. The owner whose affair has been managed is not bound to fulfill an obligation that the manager has undertaken as a prudent administrator if he repudiates the obligation upon the obligee’s demand for performance of the obligation.
E. One who assumes the management of affairs of another must exercise the care of a prudent administrator, but is answerable for any loss that results from his failure to do so only upon a showing of self-dealing, breach of fiduciary duty or bad faith.

A

C. A management of affairs is subject to the rules of mandate to the extent that those rules are compatible with management of affairs.

A is incorrect, since management of the affairs of another is a situation in which the manager acts without authority. Management of the affairs of another and unjust enrichment are two very different types of obligation, making B incorrect. D and E are wrong, as the person whose affairs were managed must fulfill the obligations the manager has undertaken and reimburse him necessary and useful expenses, while the manager is liable for loss he causes under the standard that he exercise care.

How well did you know this?
1
Not at all
2
3
4
5
Perfectly
18
Q

A relatively null contract:

A. is a contract that is considered by the law to be null because it is between relatives of the first degree of propinquity of consanguinity.
B. is a contract that violates a rule of public order.
C. is a contract that violates a rule intended for the protection of private parties.
D. is a contract the object of which is illicit or immoral.
E. may be annulled by any person in a suit for annulment brought within ten years from the time the ground for nullity either ceased or was discovered.

A

C. is a contract that violates a rule intended for the protection of private parties.

B and D describe absolute nullities. E inaccurately describes the prescriptive period; it is five years, not ten.

How well did you know this?
1
Not at all
2
3
4
5
Perfectly
19
Q

Which of the following must be in writing to be enforceable? (NOTE: Each alternative choice below should be considered a correct answer only if ALL of the types of agreements enumerated in the choice must be in writing to be enforceable)

A. The sale, lease and mortgage of an immovable.
B. A mortgage, a security agreement, and a contract having a price or value exceeding $500.
C. A suretyship, a transaction or compromise that is reached out of open court; and a conventional mortgage.
D. The sale of future things, the sale of a hope and the sale of a thing pending litigation of ownership.
E. All of the above.

A

C. A suretyship, a transaction or compromise that is reached out of open court; and a conventional mortgage.

The lease of an immovable may be oral, so A is incorrect. B is wrong because a security agreement may be oral if the collateral is in the possession or control of the creditor. There is no requirement that sales of further things or hopes be in writing, making D incorrect.

How well did you know this?
1
Not at all
2
3
4
5
Perfectly
20
Q

A sale of litigious rights:

A. is an absolute nullity.
B. is a relative nullity.
C. gives the debtor the right to extinguish his obligation by paying to the assignee twice the price the assignee paid for the assignment.
D. reduces the debtor’s obligation by the amount paid by the assignee for the assignment.
E. has none of the attributes or effects described above.

A

E. has none of the attributes or effects described above.

Sales of litigious rights may be valid. They give rise to a right in the debtor to satisfy his duty by paying to the transferee the price the transferee has been paid. Thus, choices A through D are incorrect.

How well did you know this?
1
Not at all
2
3
4
5
Perfectly
21
Q

Which of the following is a correct statement of Louisiana law?

A. Interpretation of a contract is the determination of the common intent of the parties.
B. When the words of a contract are clear and explicit and lead to no absurd consequences, interpretation in search of the parties’ intent is nonetheless appropriate when necessary to conform the parties’ agreement to prevailing usages.
C. When the parties intend a contract of general scope but, to eliminate doubt, include a provision that describes a specific situation, the proper interpretation of the contract is to restrict its scope to that situation alone.
D. In case of doubt that cannot otherwise be resolved, a contract must be interpreted against the obligor and in favor of the oblige of a particular obligation.
E. All of the above.

A

A. Interpretation of a contract is the determination of the common intent of the parties.

Choice A is a verbatim quote of Louisiana Civil Code article 2045. B is incorrect because there is no room for interpretation if the plain meaning of a contract is clear. C is incorrect because it provides precisely the opposite rule of that set out in article 2052. D is incorrect because contracts are to be construed, in cases of doubt, against the obligee.

How well did you know this?
1
Not at all
2
3
4
5
Perfectly
22
Q

Which of the following is defined by the Louisiana Civil Code to be a synallagmatic contract by which one party binds himself to give to the other party the use and enjoyment of a thing for a term in exchange for a payment that the latter party binds himself to pay?

  A. Usufruct by onerous title. 
  B. Right of use. 
  C. Lease. 
  D. Deposit. 
  E. Antichresis.
A

C. Lease.

This choice reproduces the definition of a lease from Louisiana Civil Code article 2668 exactly, except that it replaces the word “rent” with “payment,” to make the answer slightly less obvious.

How well did you know this?
1
Not at all
2
3
4
5
Perfectly
23
Q

Under the Louisiana Civil Code, which of the following is effective only if accomplished by clear and unambiguous language that is brought to the attention of the contracting parties?

A. A waiver of the vendor’s privilege.
B. A waiver of the seller’s right to assert lesion.
C. An express dissolution clause.
D. A clause that, in advance, excludes or limits the liability of one party for intentional or gross fault that causes damage to the other party.
E. None of the above.

A

E. None of the above.

A, B and C are valid whether or not called to the other party’s attention. D is not valid even if expressed in clear and unambiguous language that is brought to the other party’s attention [See La. Civ. Code art. 2004]

How well did you know this?
1
Not at all
2
3
4
5
Perfectly
24
Q

An aleatory contract is:

A. A contract under which the performance of either party’s obligation, or the extent of the performance, depends on an uncertain event.
B. A contract that does not express the true intent of the parties.
C. A contract in which the quantity of a contractual object is determined by the output of one party or the requirements of another.
D. A contract having as its object a covenant that a third person will incur an obligation or render a performance.
E. A contract for which the law supplies no special designation.

A

A. A contract under which the performance of either party’s obligation, or the extent of the performance, depends on an uncertain event.

The definition of an aleatory contract here comes directly from Louisiana Civil Code article 1912. B describes a simulation. C describes an outputs or requirements contract described in La. Civ. Code art. 1975. D describes a promesse de porte-fort. E describes an innominate contract.

How well did you know this?
1
Not at all
2
3
4
5
Perfectly
25
Q

Which of the following is not a ground for extinction of an obligation?

  A. Performance. 
  B. Novation. 
  C. Subrogation. 
  D. Remission.
  E. Confusion.
A

C. Subrogation.

Subrogation is not a method of extinguishing obligations. It merely transfers the credit-right of an obligation from one obligee to another.

How well did you know this?
1
Not at all
2
3
4
5
Perfectly
26
Q

When an immovable described as a certain and limited body is sold for a lump price with an expression of the extent of the immovable in the act of sale,

A. The expression of the measure does not give the seller the right to a proportionate increase of the price unless there is a surplus of more than one-twentieth of the extent specified in the act of sale.
B. The price must be proportionately reduced if the seller is unable to deliver the full extent specified in the act of sale.
C. The buyer must pay to the seller a proportionate supplement of the price if the extent delivered by the seller is greater than that specified in the act of sale.
D. The buyer may recede from the sale if the actual extent of the immovable sold exceeds by more than one-twentieth the extent specified in the act of sale.
E. The expression of the extent of the immovable does not give the parties any right to an increase or diminution of the price in case of surplus or shortage in the actual extent of the immovable.

A

E. The expression of the extent of the immovable does not give the parties any right to an increase or diminution of the price in case of surplus or shortage in the actual extent of the immovable.

The situation described here, a sale of a certain and limited body for a lump price with an expression of quantity describes a sale per aversionem. In such sales, there is no adjustment of price, up or down, for a discrepancy between the represented and actual extent of the immovable.

How well did you know this?
1
Not at all
2
3
4
5
Perfectly
27
Q

Which of the following is a correct statement of Louisiana law?

A. A hope may be the object of a contract of sale.
B. A lease of a thing that does not belong to the lessor is an absolute nullity.
C. A future thing may not be the object of a contract of sale.
D. The sale of a thing belonging to another is an absolute nullity.
E. The sale of a litigious right is an absolute nullity.

A

A. A hope may be the object of a contract of sale.

A hope may be sold in Louisiana. The only other tempting choice is D. The sale of a thing belonging to another is ineffective, but it is not an absolute nullity. One must remember that such a sale may become valid under the after-acquired title doctrine.

How well did you know this?
1
Not at all
2
3
4
5
Perfectly
28
Q

Delivery of the thing is essential to the effectiveness between the parties of:

  A. A sale of a movable. 
  B. A lease of a movable. 
  C. A giving in payment. 
  D. A contract of exchange. 
  E. A security interest.
A

C. A giving in payment.

Delivery is typically not required for the effectiveness of transactions between the parties. The exception in this list is the giving in payment. Its existence requires the delivery of a thing in satisfaction of a debt.

How well did you know this?
1
Not at all
2
3
4
5
Perfectly
29
Q

Which of the following matters is effective as to a third person even though not evidenced of record?

A. A matter of capacity or authority.
B. The occurrence of a suspensive or resolutory condition.
C. The exercise of an option or right of first refusal.
D. A termination of rights that depends upon the occurrence of a condition.
E. All of the above.

A

E. All of the above.

Louisiana Civil Code article 3339 expressly lists each of these items as transactions or occurrences which are effective against third parties even absent recordation.

How well did you know this?
1
Not at all
2
3
4
5
Perfectly
30
Q

Which of the following is not a method of extinction of a mortgage?

A. The extinction or destruction of the thing mortgaged.
B. Confusion as a result of the obligee’s acquiring ownership of the thing mortgaged.
C. Prescription of all the obligations that the mortgage secures.
D. The mortgagor’s tender of substitute collateral for the secured obligation.
E. Consent of the mortgagee.

A

D. The mortgagor’s tender of substitute collateral for the secured obligation.

All of the others are explicitly mentioned in Article 3319 of the Civil Code.

How well did you know this?
1
Not at all
2
3
4
5
Perfectly
31
Q

Which of the following is a correct statement of Louisiana law?

A. In the absence of contrary agreement, a mandatary is authorized to appoint a substitute.
B. If the mandatary exceeds his authority, he is answerable to the principal for resulting loss that the principal sustains.
C. A mandatary acts outside the limits of his authority when he fulfills his duties in a manner more advantageous to the principal than was authorized.
D. The principal is not bound to reimburse the mandatary for the expenses and charges he has incurred if the purpose of the mandate was not accomplished.
E. All of the above.

A

B. If the mandatary exceeds his authority, he is answerable to the principal for resulting loss that the principal sustains.

[A is incorrect because a mandatary may appoint a substitute only under circumstances which prevent the mandatary from acting personally when he is also unable to communicate with the principal. Choice A implies that the ability to appoint a substitute is the general rule. In fact, it is a very narrow exception to the rule that the mandatary must generally fulfill the mandate himself. C is not correct; a mandatary does not exceed his authority if he fulfills his duty in a way more advantageous to the principal than what was authorized. D is not correct, as the principal gets no relief from liability merely because the mandatary was unsuccessful.]

How well did you know this?
1
Not at all
2
3
4
5
Perfectly
32
Q

In a revocatory action seeking the annulment of a gratuitous contract made by an obligor, which of the following must the plaintiff prove?

A. That the contract caused or increased the obligor’s insolvency.
B. That the plaintiff has rights against the obligor upon a claim that was liquidated by a judgment prior to the execution of the contract.
C. That the other party to the contract knew that the contract would cause or increase the obligor’s insolvency.
D. That the contract was made in fraud of the plaintiff’s rights.
E. All of the above.

A

A. That the contract caused or increased the obligor’s insolvency.

[The revocatory action allows an obligee to set aside the transaction of his obligor that causes or increases the obligor’s insolvency. To bring the action, the obligee must have a right which arose before the transaction sought to be set aside. There is no requirement that the obligee’s claim be liquidated, as B suggests. If the transaction the obligee seeks to set aside is gratuitous, whether the other party to it knew it would cause or increase the obligor’s insolvency is irrelevant, so C is incorrect. Finally, D is incorrect. The “fraud of the [plaintiff’s] rights” standard was removed from the revocatory action in the 1984 revision of the Obligations articles of the Civil Code.]

How well did you know this?
1
Not at all
2
3
4
5
Perfectly
33
Q

Subrogation is:

A. an assumption by a third person of an obligation of an obligor.
B. the extinguishment of a new obligation by the substitution of a new one.
C. an agreement under which a new obligor is substituted for a prior obligor who is discharged by the obligee.
D. the substitution of one person to the rights of another.
E. the extinguishment of obligations arising by operation of law when two persons owe to each other sums of money that are liquidated and presently due.

A

D. the substitution of one person to the rights of another.

[Choice A addresses an assumption, not a subrogation. B defines a novation. C seems to describe a subjective novation. E describes legal compensation.]

How well did you know this?
1
Not at all
2
3
4
5
Perfectly
34
Q

Which of the following is not a correct statement of Louisiana law?

A. Solidary suretyship exists when a person binds himself as a principal obligor by promising that a third person will incur an obligation or render a performance.
B. Suretyship is an accessory contract by which a person binds himself to a creditor to fulfill the obligation of another upon the failure of the latter to do so.
C. Suretyship must be express and in writing.
D. A surety who pays the principal obligation is subrogated by operation of law to the rights of the creditor.
E. The surety may assert against the creditor any defense to the principal obligation that the principal obligor could assert except lack of capacity or discharge in bankruptcy.

A

A. Solidary suretyship exists when a person binds himself as a principal obligor by promising that a third person will incur an obligation or render a performance.

[B, C, D and E are all correct statements concerning suretyship. A is incorrect because it does not describe “solidary suretyship,” rather it addresses a promesse de porte-fort under La. Civ. Code art. 1977. Solidary suretyship exists by default when more than one surety has guaranteed the same debt. A is further made incorrect by its language that suretyship exists when a person binds himself as a “principal” obligor. Suretyship is an accessorial obligation and as such, a surety’s role is accessory to someone else being a principal obligor.]

How well did you know this?
1
Not at all
2
3
4
5
Perfectly
35
Q

A transfer of immovable property:

A. must be made by authentic act or by act under private signature duly acknowledged before a notary public.
B. is never valid unless in writing.
C. is valid between the parties, even though the agreement of transfer is oral, if the property has been actually delivered and the transferor recognizes the transfer when interrogated on oath.
D. has effect against third persons only from the time the instrument of transfer is filed for registry in the parish of domicile of the transferor.
E. is effective between the parties only if passed before a notary and two witnesses or entirely written, dated and signed in the handwriting of the transferor.

A

C. is valid between the parties, even though the agreement of transfer is oral, if the property has been actually delivered and the transferor recognizes the transfer when interrogated on oath.

[A transfer of immovable property must be made by an act under private signature, or some more elaborate form (such as an authentic act). However, there is an exception between the parties which makes an oral transfer recognized under oath valid where the property has been delivered. A is incorrect because an act under private signature will suffice. B is incorrect because of the word “never,” which fails to recognize the exception to the general rule, detailed above. D is incorrect because the Public Records Doctrine would require filing of an act affecting immovable property in the parish where the property is situated, not in the parish of a particular party’s domicile. E seems to bring in requirements relevant to wills, not the basic transfer of immovables.]

How well did you know this?
1
Not at all
2
3
4
5
Perfectly
36
Q

In the case of an assumption agreed upon by an obligee and an assuming obligor:

A. the agreement effects a release of the original obligor.
B. the assuming obligor may raise against the obligee any defense based on the relationship between the assuming obligor and the original obligor.
C. the assuming obligor may invoke compensation based on an obligation owed by the obligee to the original obligor.
D. the assuming obligor is subrogated by operation of law to the rights of the original obligor in any property of the latter given as security for the assumed obligation.
E. the agreement between the assuming obligor and the obligee must be made in writing.

A

E. the agreement between the assuming obligor and the obligee must be made in writing.

[An assumption between the third party (“assuming obligor”) and the obligee must always be in writing. A is incorrect because the original obligor remains bound in an assumption. The defenses described in B may not be asserted in this form of assumption. C and D simply have no basis in the law.]

How well did you know this?
1
Not at all
2
3
4
5
Perfectly
37
Q

A contract made by a person without legal capacity:

A. is absolutely null.
B. is relatively null.
C. may be rescinded at the request of any party to the contract.
D. is not susceptible of confirmation.
E. does not give rise to a natural obligation, regardless of whether the incapable person was endowed with discernment.

A

B. is relatively null.

[The Civil Code provides that contracts made by a person lacking capacity are relative nullities. C and D are incorrect, as a relative nullity may be asserted only by the person for whose interest the nullity exists and relative nullities may be confirmed. E is incorrect because the Civil Code expressly lists a contract made by an incapacitated person as an example of a scenario giving rise to a natural obligation.]

How well did you know this?
1
Not at all
2
3
4
5
Perfectly
38
Q

Which of the following is a correct statement of Louisiana law?

A. A depositary is bound to return the precise thing that he received in deposit.
B. A deposit is a contract by which a person delivers an immovable to another for safekeeping under the obligation of returning it to the depositor upon demand.
C. The formation of a contract of deposit is complete upon agreement of the thing, price and consent of the parties, regardless of whether the thing has been delivered to the depositary.
D. The depositary may use the thing deposited unless this right has been interdicted in the contract of deposit.
E. If the deposit is onerous, the depositary is responsible for loss resulting from an irresistible force.

A

A. A depositary is bound to return the precise thing that he received in deposit.

[The very nature of the contract of deposit is the safeguarding of a thing to be returned to the depositor. The contract may pertain to immovable or movable property, which makes B incorrect. The contract is not effective until delivery, which makes C incorrect. D is wrong, as the depositary may not use the deposited thing unless he has the express or implied permission of the depositor. And E is not correct, as neither onerous nor gratuitous depositaries are responsible for losses caused by a force majeure. Such forces typically extinguish obligations as a result of impossibility of performance.]

How well did you know this?
1
Not at all
2
3
4
5
Perfectly
39
Q

When an obligation is alternative,

A. the obligor is bound to multiple items of performance that must be separately rendered or enforced, each item being regarded as the object of a separate obligation.
B. the choice of the item of performance belongs to the obligee unless it has been expressly or impliedly granted to the obligor.
C. the obligor is bound to render only one of two or more items of performance.
D. the obligor may perform the obligation by rendering as performance a part of one item and a part of another.
E. the obligation is wholly extinguished if one of several items of performance contemplated in the alternative obligation becomes impossible or unlawful without the obligor’s fault.

A

C. the obligor is bound to render only one of two or more items of performance.

[Choice A describes a conjunctive obligation. B is incorrect, because the choice in an alternative obligation generally belongs to the obligor, not the obligee. D is wrong, as an alternative obligation requires the obligor to choose and render an entire performance. E is incorrect because, if some of the items of performance in an alternative obligation become impossible without obligor fault, he must render one of the performances which remain.]

How well did you know this?
1
Not at all
2
3
4
5
Perfectly
40
Q

In a lease of immovable property to be occupied as a dwelling, the lessor warrants:

A. that the thing is suitable for the purpose for which it was leased.
B. that the thing is free of vices and defects that prevent its use for the purpose for which it was leased.
C. that the lessee will have peaceful possession of the leased thing against any person who asserts ownership, or right to possession of, or any other right in the thing.
D. that the lessee will have peaceful possession of the leased thing against any person who, with the lessor’s consent, has access to the thing or occupies adjacent property belonging to the lessor.
E. All of the above.

A

E. All of the above.

[The lessor makes all of the warranties described here in a lease.]

How well did you know this?
1
Not at all
2
3
4
5
Perfectly
41
Q

A privilege arises:

A. only if granted by authentic act or act under private signature.
B. only if granted by authentic act or act under private signature duly acknowledged.
C. only if granted under a contract proved by at least one witness and other corroborating circumstances.
D. only if the act creating it is in writing; however, a privilege may be created by oral agreement if the obligor recognizes the privilege when interrogated under oath.
E. only for those debts for which it is expressly granted by law.

A

E. only for those debts for which it is expressly granted by law.

[The remaining choices are inaccurate, as no documentation is required for most privileges (authentic act or otherwise), as they arise by operation of law whenever certain acts identified by law occur, which give rise to payment obligations that are secured by the privilege.]

42
Q

Which of the following is a correct statement of Louisiana law?

A. The choice of law by the parties to a contract is enforceable only to the extent that the law chosen is the law of the state whose policies would be most seriously impaired if its law were not applied to the contract.
B. The choice of law by the parties to a contract is enforceable only if the law chosen is the law of a state in which at least one of the parties is domiciled.
C. All issues of conventional obligations are governed by the law expressly chosen by the parties, provided that the law chosen is (1) the law of the state of making, (2) the law of the state of performance or (3) the law of the state of common domicile or place of business of the parties.
D. All issues of conventional obligations are governed by the law expressly chosen by the parties.
E. None of the above.

A

E. None of the above.

[Except for issues of form and capacity, parties are free to select the law that will govern their contract (chosen law) except to the extent that their chosen law violates a strong public policy of the law that would have governed had they made no selection (lex causae). La. Civ. Code art. 3540. All four choices are incorrect. Choice A destroys party autonomy by limiting the chosen law to the lex causae. Choice B limits the chosen law to the law of the domicile of one of the parties. Choice C limits the chosen law to the laws that regulate the validity of the contract’s form. La. Civ. Code art. 3537. Choice D imposes no limitations on party autonomy.]

43
Q

A judicial mortgage is:

A. a general mortgage affecting all movable and immovable property of the obligor.
B. a general mortgage burdening only that immovable property owned by the obligor at the time that the judicial mortgage is created.
C. a general mortgage burdening the property of the obligor as well all property of his heirs or legatees who have accepted his succession.
D. a general mortgage established over all property susceptible of mortgage that the obligor owns when the judicial mortgage is created and over future property susceptible of mortgage when the obligor acquires it.
E. a special mortgage that is created by filing with the recorder of mortgages a judgment expressly declaring or recognizing that the judgment creditor has a mortgage upon property of the obligor.

A

D. a general mortgage established over all property susceptible of mortgage that the obligor owns when the judicial mortgage is created and over future property susceptible of mortgage when the obligor acquires it.

[The other choices are wrong in one small way or another; e.g., judicial mortgages do not affect movable property (A), they extend to future property (B), they extend only to property owned by the obligor (C), and they are general mortgages (E).]

44
Q

The extinction of the principal obligation:

A. extinguishes ordinary suretyship for the obligation, but does not extinguish commercial suretyship.
B. extinguishes suretyship for the obligation, but does not extinguish mortgages securing the obligation.
C. extinguishes all mortgages securing the obligation, but does not extinguish suretyship.
D. extinguishes both suretyship and mortgages securing the obligation.
E. has no effect on either suretyship or mortgages securing the obligation.

A

D. extinguishes both suretyship and mortgages securing the obligation.

[For all security devices generally (with the notable exception of the new Multiple Indebtedness Mortgage, or MIM), extinction of the obligation extinguishes the security device. Without an obligation to be secured, the security device has no role, and is thus extinguished.]

45
Q

The term of a reconducted nonagricultural lease having an initial term of one year is:

  A. from year to year. 
  B. from month to month. 
  C. from week to week. 
  D. from day to day. 
  E. for a period equal in length to the initial term of the lease.
A

B. from month to month

[This rule is from La. Civ. Code art. 2723. When the original term of a reconducted nonagricultural lease is a month or longer, the term of the reconduction is month to month. A’s answer-choice of “year to year” is the term of a reconducted agricultural lease found in La. Civ. Code art. 2722. D applies to reconducted nonagricultural leases whose term is at least a day but shorter than a month. E applies to leases where the initial term is less than a day.]

46
Q

An agreement whereby one party promises to sell and the other party promises to buy a thing at a later time is:

  A. a right of first refusal. 
  B. an option. 
  C. a bilateral promise of sale. 
  D. a sale by weight, tale or measure. 
  E. a sale on view or trial.
A

C. a bilateral promise of sale.

[This question comes verbatim from the rule stated in La. Civ. Code art. 2623.]

47
Q

Which of the following is a correct statement of Louisiana law?

A. Nullity of the principal obligation renders a stipulated damages clause null.
B. Nullity of the stipulated damage clause renders the principal obligation null.
C. An obligor whose failure to perform the principal obligation is justified by a valid excuse is nonetheless bound for the payment of stipulated damages.
D. An obligee who avails himself of a stipulated damages clause must prove that the actual damages caused by the obligor’s breach approximate the amount of the stipulated damages.
E. An obligee may avail himself of a clause stipulating damages for delay without the necessity of putting the obligor in default.

A

A. Nullity of the principal obligation renders a stipulated damages clause null.

[This question comes verbatim from the rule stated in La. Civ. Code art. 2006. That same code article clearly states that nullity of the stipulated damages clause does not render the principal obligation null so choice B is not the correct response. C is incorrect because an obligor whose failure to perform the principal obligation is justified by a valid excuse is also relieved of liability under a stipulated damages clause. La. Civ. Code art. 2008. An obligee who takes benefit of a stipulated damages clause need not prove actual damage caused by the obligor’s breach under La. Civ. Code art. 2009 so choice D is also not correct. Lastly, the obligor must be put in default before an obligee avails himself of a stipulated damages clause under La. Civ. Code art. 2010 so choice E is incorrect.]

48
Q

An obligation is rendered null by:

A. A suspensive condition that depends solely on the will of the obligor.
B. A suspensive condition that depends solely on the will of the obligee.
C. A resolutory condition that depends solely on the will of the obligor.
D. A resolutory condition that depends solely on the will of the obligee.
E. All of the above.

A

No choice is correct.

[This question addresses the issue of nullity resulting from a “potestative condition.” Here is the rule from the Code: “A suspensive condition that depends solely on the whim of the obligor makes the obligation null.” La. Civ. Code art. 1770. Choice A says “will” instead of “whim.” Choice B says “obligee” instead of “obligor.” Choice C says “resolutory condition” instead of “suspensive condition.” Choice D says “resolutory condition” instead of “suspensive condition and “obligee” instead of “obligor.”]

49
Q

Which of the following is not a correct statement of Louisiana law?

A. A contract is an agreement by two or more parties whereby obligations are created, modified or extinguished.
B. A contract is bilateral when the parties obligate themselves reciprocally so that the obligation of each party is correlative to the obligation of the other.
C. A contract is onerous when each of the parties obtains an advantage in exchange for his obligation.
D. A contract is aleatory when it is not given a special designation, such as sale, lease, loan or insurance.
E. A contract is commutative when the performance of the obligation of each party is correlative to the performance of the other.

A

D. A contract is aleatory when it is not given a special designation, such as sale, lease, loan or insurance.

[A contract is aleatory when performance by a party depends on an uncertain event. La. Civ. Code art. 1912. Choice A defines a contract. La. Civ. Code art. 1906. Choice B defines a bilateral contract. La. Civ. Code art. 1908. Choice C defines an onerous contract. La. Civ. Code art. 1909. Choice E defines a commutative contract. La. Civ. Code art. 1911.]

50
Q

Fraud vitiates consent:

A. only when it arises from an affirmative representation of the truth.
B. only if the error induced by fraud concerns the principal cause of the obligation.
C. regardless of whether the party against whom the fraud was directed could have ascertained the truth without difficulty, inconvenience, or special skill.
D. when the fraud is committed by a third person if the other party to the contract knew or should have known of the fraud.
E. only if proved by clear and convincing evidence.

A

D. when the fraud is committed by a third person if the other party to the contract knew or should have known of the fraud.

[This comes straight out of La. Civ. Code art. 1956. Choice A is not correct because fraud can arise from suppression of the truth. La. Civ. Code art. 1953. Choice B is not correct because error need not concern principal cause, though “it must concern a circumstance that has substantially influenced” consent. La. Civ. Code art. 1955. Choice C is not correct because the defrauded party has some duty to ascertain the truth unless a relation of confidence existed between the parties. La. Civ. Code art. 1954. Choice E is not correct because fraud need only be proved by a preponderance of the evidence. La. Civ. Code art. 1957.]

51
Q

When the law requires a contract to be in a written form, the contract may not be proved by testimony or by presumption unless:

A. the consent of one of the parties to the contract was vitiated by a vice of consent.
B. the written instrument has been destroyed, lost or stolen.
C. proof of the contract is established by at least one witness and other corroborating circumstances.
D. both parties to the contract admit its existence when interrogated on oath.
E. the proponent of the contract has made a valid tender of performance under the contract.

A

B. the written instrument has been destroyed, lost or stolen.

[This is a verbatim recitation of the rule set out in Louisiana Civil Code article 1832.]

52
Q

The Louisiana Civil Code lists the following vices of consent:

A. Error, fraud and duress.
B. Error, fraud and unlawful cause.
C. Error, fraud and ambiguity.
D. Error, fraud and lack of good faith.
E. Error, fraud, duress, unlawful cause, ambiguity and lack of good faith.

A

A. Error, fraud and duress.

[While good faith is a required duty in all obligations, failure to exercise it does not create a vice of consent. Likewise, unlawful cause may create a nullity, but it is not one of the vices of consent.]

53
Q

In the case of a contract of mandate that is silent on the issue of termination, which of the following events does not terminate the mandate?

A. Death of the mandatary.
B. Interdiction of the mandatary.
C. Death of the principal.
D. Incapacity or disability of the principal.
E. Qualification of the curator after the interdiction of the principal.

A

D. Incapacity or disability of the principal.

[Death and interdiction generally terminate a mandate. There is no provision for termination of mandate for lesser grounds such as incapacity or disability.]

54
Q

Which of the following is an incorrect statement of Louisiana law?

A. A contract is formed by the consent of the parties established through offer and acceptance.
B. Unless otherwise specified in the offer, there must be conformity between the manner in which the offer is made and the manner in which the acceptance is made.
C. Unless the law prescribes a certain formality for the intended contract, offer and acceptance may be made orally, in writing, or by action or inaction that under the circumstances is clearly indicative of consent.
D. An offer expires by the death or incapacity of the offeror or the offeree before it has been accepted.
E. A revocable offer expires if not accepted within a reasonable time.

A

B. Unless otherwise specified in the offer, there must be conformity between the manner in which the offer is made and the manner in which the acceptance is made.

[While the terms of the acceptance must mirror the terms of the offer, there is no requirement that an acceptance be made in the same manner as the offer. The only requirement is that acceptance, in order to benefit from the rule allowing it to be effective on transmission, be made in the manner and medium suggested by the offer or in a reasonable manner and by a reasonable medium.]

55
Q

A contract made by a person without legal capacity is:

A. relatively null and may be rescinded only at the request of that person or his legal representative.
B. relatively null and may be rescinded at the request of any person or by the court on its own initiative.
C. absolutely null and may be rescinded only at the request of that person or his legal representative.
D. insusceptible of confirmation or ratification.
E. is an example of a contract that violates a rule of public order.

A

A. relatively null and may be rescinded only at the request of that person or his legal representative.

[A is the best answer here. Louisiana Civil Code article 2031 describes a contract in which a party lacked capacity as a relative nullity. Such contracts may be confirmed and may only be invoked by the person in whose interest the ground for nullity exists. You may recall from law school that some civil law doctrine and jurisprudence categorizes certain extreme incapacities (so called “incapacities of enjoyment”) absolute nullities, but for bar examination purposes and to interpret this question in such a way that allows for the existence of a correct answer, you should focus on article 2031 of the Louisiana Civil Code.]

56
Q

A stipulation for the benefit of a third party beneficiary:

A. may still be dissolved by the parties to the contract by mutual consent without the beneficiary’s agreement, even if the beneficiary has manifested his intention to avail himself of the benefit.
B. deprives the promisor of the ability to raise against the beneficiary such defenses based on the contract as he may have raised against the stipulator.
C. precludes the stipulator from demanding performance from the promisor for the benefit of the beneficiary.
D. gives the beneficiary the right to demand performance from the promisor.
E. excuses the promisor from the obligation to render performance to the stipulator in cases where the stipulation is revoked or refused.

A

D. gives the beneficiary the right to demand performance from the promisor.

[This is the essential feature of a third party beneficiary contract. The third party beneficiary is given a right to enforce the contract against the promisor even though there is no privity and absent special third party beneficiary rules, the third party would not be able to enforce a contract to which he is not a party.]

57
Q

According to the Louisiana Civil Code, a contract of unspecified duration:

A. is unenforceable in all cases.
B. is unenforceable, unless the contract has been ratified or confirmed by both parties.
C. is relatively null.
D. is presumed to have a duration from month to month.
E. may be terminated at the will of either party by giving notice, reasonable in time and form, to the other party.

A

E. may be terminated at the will of either party by giving notice, reasonable in time and form, to the other party.

[This is a verbatim recitation of the rule set out in Louisiana Civil Code article 2024.]

58
Q

An obligor in good faith who fails to perform a conventional obligation is liable:

A. only for the damages that were foreseeable at the time the contract was made.
B. only for moratory damages.
C. for all damages, foreseeable or not, that are a direct consequence of his failure to perform.
D. for all damages caused by his failure to perform, including damages for nonpecuniary loss, regardless of the nature of the contract.
E. for damages that might have been avoided if the obligee had taken reasonable efforts to mitigate the damage caused by the obligor’s failure to perform.

A

A. only for the damages that were foreseeable at the time the contract was made.

[The damages owed by an obligor in good faith are substantially limited under Louisiana Civil Code article 1996.]

59
Q

A legal suretyship is a suretyship:

A. that does not violate a rule of public order.
B. that is given pursuant to legislation, administrative act or regulation, or a court order.
C. in which the principal obligor is a business corporation, partnership, or other business entity.
D. in which the surety ostensibly binds himself as a principal obligor to satisfy the present and future obligations of another.
E. in which the surety has encumbered his property as security for the obligation of another but has not personally bound himself.

A

B. that is given pursuant to legislation, administrative act or regulation, or a court order.

[A “legal” suretyship is one created by law, as described in this option. A is simply wrong, as it construes the word “legal” too broadly. C is wrong because this is the definition of a “commercial” suretyship. D is wrong because this is an “ostensible” suretyship if the obligee knows that this is the principal cause of the surety’s binding himself. E is wrong because this is a description of an “in rem” or “non-recourse” mortgage or security interest.]

60
Q

The recordation of an instrument transferring an immovable:

A. causes the instrument to constitute full proof of the agreement it contains as against the parties, their heirs and successors by universal or particular title.
B. charges third persons with knowledge of the instrument and the recitations contained therein.
C. is necessary in order for the instrument to have effect against third persons.
D. creates a presumption that the instrument is valid and genuine.
E. creates a presumption as to the capacity of the parties.

A

C. is necessary in order for the instrument to have effect against third persons.

[This choice correctly describes Louisiana’s Public Records Doctrine. See also Louisiana Civil Code article 1839. Knowledge of an instrument is irrelevant. Mere recordation is sufficient to bind third parties.]

61
Q

Which of the following is a correct statement of the law?

A. The action in redhibition against a bad faith seller prescribes in four years from the day delivery of the thing sold was made to the buyer or one year from the day the defect was discovered by the buyer, whichever occurs first.
B. The seller is presumed to have known of a redhibitory defect if it appears within three days from the time of delivery of the thing sold.
C. In an action for rescission because of a redhibitory defect, the court may limit the remedy of the buyer to reduction of the purchase price.
D. An exclusion or limitation of the warranty against redhibitory vices is not permitted in consumer transactions.
E. All of the above.

A

C. In an action for rescission because of a redhibitory defect, the court may limit the remedy of the buyer to reduction of the purchase price.

[Choice B is tricky. The rule is that a redhibitory defect which appears within 3 days of delivery is presumed to have existed at delivery. This has no bearing on the seller’s knowledge of the defect, however.]

62
Q

Which of the following is an example of circumstances under which a natural obligation arises?

A. A contract to sell immovable property is unenforceable because it is not in writing.
B. A borrower enters into an unenforceable agreement to pay interest at a usurious rate.
C. A civil obligation has been extinguished by prescription.
D. A contract is unenforceable because it is predicated upon unlawful cause.
E. All of the above.

A

C. A civil obligation has been extinguished by prescription.

[Choice C describes the classic example of a natural obligation, and one source of it expressly mentioned in Louisiana Civil Code article 1762. Most of the other examples represent contracts which the law regards as invalid for reasons of general public policy — contracts without lawful case, which may include gambling debts, and contracts involving usurious interest — and therefore the law implies no moral duty.]

63
Q

A synallagmatic contract is a contract

A. that is entirely oral.
B. that is given no special designation, such as a designation as a sale, lease, loan or insurance.
C. under which the parties obligate themselves reciprocally.
D. that contains an obligation subject to a condition that depends solely on the whim of the obligor.
E. that is made by a party without contractual capacity.

A

C. under which the parties obligate themselves reciprocally.

[The Louisiana Civil Code uses the phrase “synallagmatic contract” to describe a lease. It is a bilateral contract. Each party is bound to render a performance. Response B describes an innominate contract and D describes a contract with a potestative condition. Response A is incorrect because whether a contract is written or oral has no bearing on its classification as synallagmatic.]

64
Q

In a revocatory action seeking the annulment of a gratuitous contract made by an obligor, which of the following must the plaintiff prove?

A. that the contract caused or increased the obligor’s insolvency.
B. that the plaintiff has rights against the obligor upon a claim that was liquidated by a judgment prior to the execution of the contract.
C. that the other party to the contract knew that the contract would cause or increase the obligor’s insolvency.
D. that the contract was made in fraud of the plaintiff’s rights.
E. all of the above.

A

A. that the contract caused or increased the obligor’s insolvency.

[The revocatory action allows an obligee to set aside the transaction of his obligor that causes or increases the obligor’s insolvency. To bring the action, the obligee must have a right which arose before the transaction sought to be set aside. There is no requirement that the obligee’s claim be liquidated, as B suggests. If the transaction the obligee seeks to set aside is gratuitous, whether the other party to it knew it would cause or increase the obligor’s insolvency is irrelevant, so C is incorrect. Finally, D is incorrect. The “fraud of the [plaintiff’s] rights” standard was removed from the revocatory action in the 1984 revision of the Obligations articles of the Civil Code.]

65
Q

A mandate necessarily terminates upon

A. any condition that makes express revocation of the mandate impossible or impractical.
B. the filing of a revocatory action against the principal.
C. incapacity of the principal.
D. death of the principal.
E. the expiration of ten years from the date the mandate was granted.

A

D. death of the principal.

[Mandate generally terminates upon the death of the principal. This answer is not perfect, as the mandatary must still complete undertakings he began before the principal’s death if delay would injure the principal’s interest. But it is the “best” answer here. Note that response C is incorrect because not all incapacities of the principal terminate the mandate; interdiction does, but there may be other causes of the principal’s incapacity.]

66
Q

A natural obligation:

A. is not onerous cause for a promise to fulfill it.
B. arises from circumstances in which the law implies a particular moral duty to render a performance.
C. is enforceable by judicial action.
D. arises, for example, when a party agrees to a performance that is contra bonos mores.
E. has all of the characteristics and effects described above.

A

B. arises from circumstances in which the law implies a particular moral duty to render a performance.

[Choice B uses the exact codal definition of a natural obligation. D is incorrect because a natural obligation exists only when the law implies a moral duty, not when a party agrees to an immoral performance (a performance contra bonos mores). C and A are incorrect because, in contrast with civil obligations, natural obligations are not judicially enforceable, yet promises to fulfill them will amount to onerous cause.]

67
Q

Which of the following is a correct statement of the law?

A. Novation is the extinguishment of an existing obligation by the substitution of a new one.
B. Objective novation takes place when a new obligor is substituted for a prior obligor who is discharged by the obligee.
C. A novation made by the obligee and one of the obligors of a solidary obligation does not effect a release of the other solidary obligors.
D. Execution of a new writing and modification of an obligation, made without intention to extinguish it, are examples of novation.
E. All of the above.

A

A. Novation is the extinguishment of an existing obligation by the substitution of a new one.

[Choice A memorializes the codal definition of a novation. B describes a subjective novation. C is incorrect, because novation of a solidary obligation releases all solidary obligors. D describes “mere modifications,” as distinguished from a novation.]

68
Q

Which of the following is a correct statement of the law?

A. There is a management of affairs when a person, the principal, confers authority on another person, the manager, to manage the principal’s affairs or otherwise act to protect the principal’s interests.
B. Management of affairs is synonymous with unjust enrichment.
C. The owner whose affair has been managed is not bound to fulfill an obligation that the manager has undertaken as a prudent administrator if he repudiates the obligation upon the obligee’s demand for performance of the obligation.
D. A management of affairs is subject to the rules of mandate to the extent that those rules are compatible with management of affairs.
E. One who assumes the management of the affairs of another must exercise the care of a prudent administrator, but is answerable for any loss that results from his failure to do so only upon a showing of self-dealing, breach of fiduciary duty or bad faith.

A

D. A management of affairs is subject to the rules of mandate to the extent that those rules are compatible with management of affairs.

[A is incorrect, since management of the affairs of another is a situation in which the manager acts without authority. Management of the affairs of another and unjust enrichment are two very different types of obligation, making B incorrect. C and E are wrong, as the person whose affairs were managed must fulfill the obligations the manager has undertaken and reimburse him necessary and useful expenses, while the manager is liable for loss he causes under the standard that he exercise care.]

69
Q

Which of the following is not a ground for extinction of an obligation?

  A. performance.
  B. novation. 
  C. subrogation. 
  D. remission. 
  E. confusion.
A

C. subrogation.

[Subrogation is not a method of extinguishing obligations. It merely transfers the credit-right of an obligation from one obligee to another.]

70
Q

A sale of litigious rights:

A. is an absolute nullity.
B. is an relative nullity.
C. gives the debtor the right to extinguish his obligation by paying to the assignee twice the price the assignee paid for the assignment.
D. reduces the debtor’s obligation by the amount paid by the assignee for the assignment.
E. has none of the attributes or effects described above.

A

E. has none of the attributes or effects described above.

[Sales of litigious rights may be valid. They give rise to a right in the debtor to satisfy his duty by paying to the transferee the price the transferee has been paid. Thus, choices A through D are incorrect.]

71
Q

Which of the following is a correct statement of Louisiana law?

A. Interpretation of a contract is the determination of the common intent of the parties.
B. When the words of a contract are clear and explicit and lead to no absurd consequences, interpretation in search of the parties’ intent is nonetheless appropriate when necessary to conform the parties’ agreement to prevailing usages.
C. When the parties intend a contract of general scope but, to eliminate doubt, include a provision that describes a specific situation, the proper interpretation of the contract is to restrict its scope to that situation alone.
D. In case of doubt that cannot otherwise be resolved, a contract must be interpreted against the obligor and in favor of the obligee of a particular obligation.
E. All of the above.

A

A. Interpretation of a contract is the determination of the common intent of the parties.

[Choice A is a verbatim quote of Louisiana Civil Code article 2045. B is incorrect because there is no room for interpretation if the plain meaning of a contract is clear. C is incorrect because it provides precisely the opposite rule of that set out in article 2052. D is incorrect because contracts are to be construed, in cases of doubt, against the obligee.]

72
Q

An aleatory contract is:

A. a contract that does not express the true intent of the parties.
B. a contract in which the quantity of a contractual object is determined by the output of one party or the requirements of another.
C. a contract having as its object a covenant that a third person will incur an obligation or render a performance.
D. a contract for which the law supplies no special designation.
E. a contract under which the performance of either party’s obligation, or the extent of the performance, depends on an uncertain event.

A

E. a contract under which the performance of either party’s obligation, or the extent of the performance, depends on an uncertain event.

[The definition of an aleatory contract here comes directly from Louisiana Civil Code article 1912. A describes a simulation. B describes an outputs or requirements contract described in La. Civ. Code art. 1975. C describes a promesse de porte-fort. D describes an innominate contract.]

73
Q

The three types of suretyship listed in the Louisiana Civil Code are:

A. real suretyship, strictly personal suretyship and heritable suretyship.
B. onerous suretyship, remunerative suretyship and gratuitous suretyship.
C. commercial suretyship, legal suretyship and ordinary suretyship.
D. conventional suretyship, legal suretyship and judicial suretyship.
E. civil suretyship, natural suretyship and moral suretyship.

A

C. commercial suretyship, legal suretyship and ordinary suretyship.

[This is a simple terminology test. Note that the incorrect terms often relate to other concepts, such those in D, which are the three types of mortgages.]

74
Q

The term “collateral mortgage” is:

A. a name given to a mortgage securing an obligation evidenced by a writing that is issued, pledged or otherwise used as security for another obligation.
B. a name given to any mortgage that is issued or otherwise used as security for an obligation of the mortgagor or a third person.
C. a name given to any mortgage that is issued or otherwise used as security for an obligation of a third person.
D. a name given to any mortgage that is established by contract.
E. a name given to any mortgage that bears upon movables.

A

A. a name given to a mortgage securing an obligation evidenced by a writing that is issued, pledged or otherwise used as security for another obligation.

[The other answers are clearly wrong. B and C are essentially the same, and a mortgage to secure a third person’s obligation is called an “in rem” mortgage. D is a “conventional” mortgage, and E is a contradiction in terms, as mortgages encumber immovables; movables are encumbered by security interests.]

75
Q

Which of the following is not a correct statement of Louisiana law?

A. A term for the performance of an obligation may be express or may be implied by the nature of the contract.
B. Performance of an obligation not subject to a term is due immediately.
C. A term for the performance of an obligation is a period of time that can be either certain or fixed.
D. A term is presumed to benefit the obligee unless the agreement or the circumstances show that it was intended to benefit the obligor or both parties.
E. A party for whose exclusive benefit a term has been established may renounce it.

A

D. A term is presumed to benefit the obligee unless the agreement or the circumstances show that it was intended to benefit the obligor or both parties.

[The precise opposite is true. Louisiana Civil Code article 1779 provides that a term is presumed to benefit the obligor unless the agreement or the circumstances show that it was intended to benefit the obligee or both parties. Generally a term will benefit the obligor by giving him more time to muster the resources to perform.]

76
Q

An obligation is solidary for the obligors:

A. when different obligors owe together just one performance to the obligee, but neither is bound for the whole.
B. only when the obligation derives from the same source for each obligor.
C. when each obligor is liable for the whole performance.
D. when an obligor is bound to render only one of two or more items of performance.
E. when each of different obligors owes a separate performance to one obligee.

A

C. when each obligor is liable for the whole performance.

[Choice C represents the Code’s definition of solidarity in article 1794. That article defines a solidary obligation, from the obligors’ perspective, as one in which each obligor is liable for the whole performance and a performance rendered by one of the solidary obligors relieves the others of liability to the obligee.]

77
Q

Subrogation occurs when:

A. one person is substituted to the legal rights of another.
B. an obligor and a third person agree to an assumption by the latter of an obligation of the former.
C. two persons owe to each other sums of money or quantities of fungible things identical in kind, and these sums or quantities are liquidated and presently due.
D. by agreement of the parties, a new performance is substituted for that previously owed.
E. a new obligor is substituted for a prior obligor who is discharged by the obligee.

A

A. one person is substituted to the legal rights of another.

[Choice C describes compensation. Choices D and E describe novation. Louisiana Civil Code article 1825 defines subrogation as the substitution of one person to the rights of another either by contract or by law.]

78
Q

The warranty against redhibitory defects:

A. is implied in every sale and may always be invoked by the buyer even if he has agreed to an exclusion or limitation of the warranty.
B. obligates a good faith seller, upon breach of the warranty, to return the purchase price with interest, reasonable expenses occasioned by the sale, damages and reasonable attorney’s fees.
C. exists only in sales of corporeals immovables.
D. applies only to defects that exist at the time of delivery.
E. applies only to defects that appear within three days after delivery.

A

D. applies only to defects that exist at the time of delivery.

[Louisiana Civil Code article 2530 requires that defects exist at the time of delivery to be redhibitory. The three-day period described in Choice E creates a presumption that the defect existed at delivery, but it is only a presumption.]

79
Q

A remission of debt:

A. may be express or tacit.
B. is presumed to exist when the obligee voluntarily surrenders to the obligor the instrument evidencing the obligation.
C. that is granted to the principal obligor releases the surety.
D. is effective when the obligor receives communication of remission from the obligee.
E. has all of the attributes and effects mentioned above.

A

E. has all of the attributes and effects mentioned above.

[Remission need not be express, and it is presumed in the situations described in Choice B, including, for instance, when the obligee surrenders a note evidencing the debt to the obligor. Choice C is an application of the general rule that extinction of the principal obligation extinguishes accessory obligations. Choice D memorializes the principle that a remission is effective only on receipt.]

80
Q

A deposit is:

A. a contract by which a person delivers consumable things to another, who is given permission to consume the things and who binds himself to return an equal amount of things of the same kind and quality.
B. a contract by which a person delivers a movable thing to another for safekeeping under the obligation of returning it upon demand.
C. a gratuitous contract by which a person delivers a nonconsumable thing to another for him to use and return.
D. a contract by which one of the parties conveys and cedes a thing to another, stipulating that the latter shall hold it as owner but reserving to the former an annual rent of a certain sum of money.
E. a contract by which a creditor acquires the right of reaping the fruits or other revenues of a thing given to him in pledge, on condition of deducting the fruits and revenues from the debt due to him.

A

B. a contract by which a person delivers a movable thing to another for safekeeping under the obligation of returning it upon demand.

[Article 2926 of the Louisiana Civil Code defines a deposit as a contract by which the depositor delivers a movable thing to a depositary for safekeeping under the obligation of returning it to the depositor on demand. Choice A describes a loan for consumption. Choice C describes a loan for use.]

81
Q

A donation inter vivos that concerns an immovable and that lacks proper form:

A. may be confirmed by the donor, but the confirmation must be made in the form required for a donation.
B. may be confirmed by the donor, but the confirmation must be made by authentic act or act under private signature duly acknowledged.
C. is not susceptible of confirmation.
D. is nonetheless valid if the property has been actually delivered and the donee recognizes the transfer when interrogated under oath.
E. is nonetheless valid if the donee proves the donation by the testimony of at least one witness and other corroborating circumstances.

A

A. may be confirmed by the donor, but the confirmation must be made in the form required for a donation.

[Louisiana Civil Code article 1845 provides that donations which are null for lack of proper form may be confirmed by the donor, but only of the confirmation is made in the form required for a donation. Choice D is tempting, as article 1839 provides that even oral transfers of immovable property may be valid between the parties, but that article requires recognition under oath by the donor/transferor, not the donee.]

82
Q

According to the Louisiana Civil Code, liberative prescription is:

A. one of the modes of extinguishment of obligations.
B. a period of time fixed by law for the existence of a right.
C. a mode of extinction of a real right other than ownership as a result of failure to exercise the right for a period of time.
D. a mode of barring actions as a result of inaction for a period of time.
E. all of the above.

A

D. a mode of barring actions as a result of inaction for a period of time.

[This definition comes straight from Louisiana Civil Code article 3447. Choice B is the definition of peremption. Choice C describes prescription of nonuse.]

83
Q

A resolutory condition:

A. prevents the enforcement of an obligation until an uncertain event occurs.
B. causes an obligation to come to an end when an uncertain event occurs.
C. that depends solely on the whim of the obligor makes the obligation null.
D. is implied in every synallagmatic contract.
E. is satisfied when a corporate officer produces authentic evidence of a resolution authorizing his execution of a contract.

A

B. causes an obligation to come to an end when an uncertain event occurs.

[Choice A describes a suspensive condition, and choice C describes the rule of nullity for suspensive conditions only. D is incorrect, as synallagmatic (in essence, bilateral) contracts need not be conditional ones.]

84
Q

In a revocatory action seeking the annulment of a gratuitous contract made by an obligor, which of the following must the plaintiff prove?

A. That the contract caused or increased the obligor’s insolvency.
B. That the plaintiff has rights against the obligor upon a claim that was liquidated by a judgment prior to the execution of the contract.
C. That the other party to the contract knew that the contract would cause or increase the obligor’s insolvency.
D. That the contract was made in fraud of the plaintiff’s rights.
E. All of the above.

A

A. That the contract caused or increased the obligor’s insolvency.

[The revocatory action allows an obligee to set aside the transaction of his obligor that causes or increases the obligor’s insolvency. To bring the action, the obligee must have a right which arose before the transaction sought to be set aside. There is no requirement that the obligee’s claim be liquidated, as B suggests. If the transaction the obligee seeks to set aside is gratuitous, whether the other party to it knew it would cause or increase the obligor’s insolvency is irrelevant. Finally, D is incorrect. The “fraud of the (plaintiff’s) rights” standard was removed from the revocatory action in the 1984 revision of the Obligations articles of the Civil Code.]

85
Q

Which of the following is an example of circumstances under which a natural obligation arises?

A. A contract to sell immovable property is unenforceable because it is not in writing.
B. A borrower enters into an unenforceable agreement to pay interest at a usurious rate.
C. A contract is unenforceable because it is predicated upon unlawful cause.
D. A civil obligation has been extinguished by prescription.
E. All of the above.

A

D. A civil obligation has been extinguished by prescription.

[Choice D describes the classic example of a natural obligation, and one source of it expressly mentioned in Louisiana Civil Code article 1762. Most of the other examples represent contracts which the law regards as invalid for reasons of general public policy — contracts without lawful case, which may include gambling debts, and contracts involving usurious interest — and therefore the law implies no moral duty.]

86
Q

Remission of a debt:

A. is effective only when express.
B. granted to a surety releases the principal obligor and all other sureties.
C. granted to one surety releases the other sureties only to the extent of the contribution the other sureties might have recovered from the surety to whom the remission was granted.
D. is not effective until the obligor has expressly accepted the remission.
E. is presumed to arise from the release of a real security given for performance of the debt.

A

C. granted to one surety releases the other sureties only to the extent of the contribution the other sureties might have recovered from the surety to whom the remission was granted.

[The other responses are clearly wrong. A remission may be tacit, and acceptance is presumed. Note that E is not entirely accurate, as releasing real security releases a commercial surety only to the extent that the release actually injures the surety, and the “performance of the debt” might have been partial.]

87
Q

Which of the following is a correct statement of Louisiana law?

A. A depositary is bound to return the precise thing that he received in deposit.
B. A deposit is a contract by which a person delivers an immovable to another for safekeeping under the obligation of returning it to the depositor upon demand.
C. The formation of a contract of deposit is complete upon agreement of the thing, price and consent of the parties, regardless of whether the thing has been delivered to the depositary.
D. The depositary may use the thing deposited unless this right has been interdicted in the contract of deposit.
E. If the deposit is onerous, the depositary is responsible for loss resulting from an irresistible force.

A

A. A depositary is bound to return the precise thing that he received in deposit.

[The very nature of the contract of deposit is the safeguarding of a thing to be returned to the depositor. The contract may pertain to immovable or movable property, which makes B incorrect. The contract is not effective until delivery, which makes C incorrect. D is wrong, as the depositary may not use the deposited thing unless he has the express or implied permission of the depositor. And E is not correct, as neither onerous nor gratuitous depositaries are responsible for losses caused by a force majeure. Such forces typically extinguish obligations as a result of impossibility of performance.]

88
Q

Subrogation is:

A. an assumption by a third person of an obligation of an obligor.
B. the substitution of one person to the rights of another.
C. the extinguishment of a new obligation by the substitution of a new one.
D. an agreement under which a new obligor is substituted for a prior obligor who is discharged by the obligee.
E. the extinguishment of obligations arising by operation of law when two persons owe to each other sums of money that are liquidated and presently due.

A

B. the substitution of one person to the rights of another.

[Choice A addresses an assumption, not a subrogation. C defines a novation. D seems to describe a subjective novation. E describes legal compensation.]

89
Q

A transfer of immovable property:

A. must be made by authentic act or by act under private signature duly acknowledged before a notary public.
B. is never valid unless in writing.
C. has effect against third persons only from the time the instrument of transfer is filed for registry in the parish of domicile of the transferor.
D. is valid between the parties, even though the agreement of transfer is oral, if the property has been actually delivered and the transferor recognizes the transfer when interrogated on oath.
E. All of the above.

A

D. is valid between the parties, even though the agreement of transfer is oral, if the property has been actually delivered and the transferor recognizes the transfer when interrogated on oath.

[A transfer of immovable property must be made by an act under private signature, or some more elaborate form (such as an authentic act). However, there is an exception between the parties which makes an oral transfer recognized under oath valid where the property has been delivered. A is
incorrect because an act under private signature will suffice. B is incorrect because of the word “never,” which fails to recognize the exception to the general rule, detailed above. C is incorrect because the Public Records Doctrine would require filing of an act affecting immovable property in the parish where the property is situated, not in the parish of a particular party’s domicile.]

90
Q

The extinction of the principal obligation:

A. extinguishes ordinary suretyship for the obligation, but does not extinguish commercial suretyship.
B. extinguishes ordinary suretyship for the obligation, but does not extinguish mortgages securing the obligation.
C. extinguishes all mortgages securing the obligation, but does not extinguish suretyship.
D. extinguishes both suretyship and mortgages securing the obligation.
E. has no effect on either suretyship or mortgages securing the obligation.

A

D. extinguishes both suretyship and mortgages securing the obligation.

[Mortgage and suretyship are both security rights, accessory contracts ancillary to the principal obligation; therefore, extinction of the principal obligation extinguishes both of these accessory contracts.]

91
Q

Which of the following is essential to the establishment of a conventional mortgage between the parties?

A. a statement that a promissory note(s) secured by the mortgage has (have) been paraphed for identification with the mortgage.
B. the signatures of both mortgagor and mortgagee.
C. filing of the mortgage in the mortgage records of the parish in which the property is located.
D. execution of the mortgage in authentic form.
E. None of the above.

A

E. None of the above.

[None of these is required for validity of the mortgage between the parties. (A) paraphs are never required; (B) only the signature of the mortgagor is required, (C) filing is necessary for effectiveness against third parties, but not between the parties; an (D) an authentic act is not required, though it is useful for evidentiary purposes (to make a “self-proving” document).]

92
Q

Which of the following is not a correct statement of Louisiana law?

A. Suretyship is an accessory contract by which a person binds himself to a creditor to fulfill the obligation of another upon the failure of the latter to do so.
B. Suretyship must be express and in writing.
C. A surety who pays the principal obligation is subrogated by operation of law to the rights of the creditor.
D. The surety may assert against the creditor any defense to the principal obligation that the principal obligor could assert except lack of capacity or discharge in bankruptcy.
E. Solidary suretyship exists when a person binds himself as a principal obligor by promising that a third person will incur an obligation or render a performance.

A

E. Solidary suretyship exists when a person binds himself as a principal obligor by promising that a third person will incur an obligation or render a performance.

[The remaining statements are perfect statements of the law, and “solidary suretyship” arises when multiple sureties together undertake the obligation described in (A), not the odd statement in (E).]

93
Q

((BARBRI))
On May 1, Daniel applies for a $10,000 loan from Bank A. Daniel signs a financing statement and a security agreement granting a security interest in Daniel’s equipment. On May 2, after checking the files and finding no competing interests, Bank A files the financing statement. On May 3, Daniel borrows $20,000 from Bank B, giving Bank B a security interest in the same equipment. Bank B loans the money and immediately files a financing statement covering the equipment. On May 10, Bank A loans Daniel $10,000 from the loan application he submitted on May 1. Who has priority to the equipment?

(a) Bank A
(b) Bank B
(c) Neither
(d) If a conflict over priority exists, Banks A and B must divide their interests.

A

(A) Bank A

When there are conflicting perfected security interests in the same collateral, priority goes to whichever party was the first to either file or perfect. Although perfection occurs upon the actual lending of the money to Daniel, Bank A has priority to the equipment under the “first to file or perfect” rule since Bank A was the first to file, notwithstanding the fact that Bank B filed and perfected its security interest before Bank A perfected its security interest.

94
Q

((BARBRI))
On April 1, Debtor authenticates a security agreement granting to A a security interest in all of Debtor’s existing and after-acquired “inventory.” That same day A files a financing statement covering the inventory. On May 1, Debtor authenticates a security agreement granting B a security interest in all of Debtor’s existing and future “accounts.” That same day B files a financing statement covering the accounts. On June 1, Debtor sells inventory to a customer on 30-day unsecured credit basis. Which secured party, A or B, has a first priority security interest in the amount owed the Debtor by the purchaser of inventory?

(a) Neither A nor B has a security interest in the amount owed.
(b) A, because it filed its financing statement before B did.
(c) B, because A’s security agreement mentioned only “inventory,” which does not encompass the amount owed.
(d) B, because A’s financing statement mentioned only “inventory,” which does not encompass the amount owed.

A

(B) A, because it filed its financing statement before B did.

A has the first-priority interest. The “amount owed” is an “account” by definition (a monetary obligation arising from sale of property), and both creditors have an interest in this account (B because its security interest specifically encompassed “accounts,” and A because the account arose upon the disposition of its original collateral, the inventory, and is thus “proceeds,” to which a security interest automatically extends). Thus, options (A) and (C) are wrong. (D) is wrong because A’s interest in the account is perfected automatically as a matter of law because (i) the original interest in inventory was perfected by filing, (ii) an interest in accounts can be perfected by filing, and (iii) the account was not acquired with cash proceeds. As between two perfected secured creditors, the first to file wins, so (B) is correct.

95
Q

((BARBRI))
Debtor is a seller of ping-pong tables, dart boards, and other recreational equipment. On February 1, Creditor lent Debtor $20,000 for expansion of Debtor’s business, and Debtor authenticated a security agreement granting Creditor an interest in all of the items on its showroom floor, now and in the future. Creditor immediately filed a financing statement
describing the collateral as Debtor’s “equipment.” On June 1, Debtor defaulted on Creditor’s loan. Does Creditor have an enforceable interest in several ping-pong tables and other recreational equipment currently on Debtor’s showroom floor?

(a) No, because Debtor holds these items for sale.
(b) No, because Creditor was not authorized to file its financing statement.
(c) Yes, because Creditor has a properly perfected security interest in Debtor’s equipment.
(d) Yes, because Creditor has fulfilled the steps for attachment of its security interest.

A

(D) Yes, because Creditor has fulfilled the steps for attachment of its security interest.

(D) is correct because, despite the problems with perfection of Creditor’s interest, Creditor has fulfilled the steps for attachment of its interest (value has been given, Debtor has rights in the collateral, and Debtor authenticated a security agreement reasonably describing the collateral — and even if the description seemed arguably too vague, the other responses are simply wrong). Creditor’s interest is therefore enforceable against Debtor, even if it would not be enforceable against third parties (of which there is no mention in this problem). (C) is wrong because Creditor does not have a properly perfected interest in Debtor’s equipment, for the reason implied in response (A): since Debtor holds this “equipment” for sale, it is actually categorized as “inventory,” so Creditor’s description of “equipment” in its financing statement is wrong and therefore fails to perfect its interest in Debtor’s showroom items. This, however, is irrelevant, because the question concerns enforcement against Debtor, not third parties. Similarly in (B), even though the financing statement describes different collateral than the security agreement, and therefore without Debtor’s authenticated permission, the filing is arguably unauthorized, that again is irrelevant with respect to enforcement against Debtor, as opposed to third parties.

96
Q

((BARBRI))
n May 1, 2013, Borrower signed an act of mortgage encumbering a parcel of Borrower’s immovable property in East Baton Rouge Parish, which was identified by a full legal description. The mortgage recited that it secured “any and all indebtedness owing from Borrower to Bank from time to time, at any time, up to a maximum of $300,000 outstanding at
any given time.” Though the act of mortgage contained a space for a signature of a Bank representative, that blank was never filled in. Moreover, the notary’s signature was not accompanied by an official seal. Is Bank’s mortgage valid and enforceable against Borrower?

(a) No, because the mortgage described the secured indebtedness inadequately.
(b) No, because the mortgage was not signed by the proper parties.
(c) No, because the mortgage was not properly notarized.
(d) Yes, because the mortgage meets the minimum form requirements imposed by law.

A

(D) Yes, because the mortgage meets the minimum form requirements imposed by law.

(D) is correct, as this mortgage is in writing, signed by the mortgagor (Borrower), and describes the secured debt and the encumbered immovable property sufficiently. (A) is wrong because this “multiple-indebtedness mortgage” mentions a maximum amount of debt to be secured at any given time, even though it is not confined to one secured loan or a fixed time period. (B) is wrong because the mortgagee (Bank) need not sign a mortgage. (C) is wrong because an act under private signature is the only legal requirement for validity; an authentic act is not required for validity. These distracters are commonly tested on the Louisiana bar exam.

97
Q

((BARBRI))
On March 1, 2003, Borrower signed a promissory note promising to repay a $120,000 loan from Bank. The note recited that Borrower would make 119 monthly installment payments of only the interest accruing on the $120,000 principal, followed by one final balloon payment of the $120,000 principal on March 1, 2013. Borrower also signed an act of mortgage, also dated March 1, 2003, encumbering a parcel of Borrower’s immovable property in Caddo Parish to
secure repayment to Bank of “that certain $120,000 promissory note dated March 1, 2003.” On August 1, 2003, Bank filed the act of mortgage for record in the Caddo Parish mortgage records. Ten years later, Borrower had failed to make several interest installments and the final balloon payment. As of August 15, 2013, is Bank’s mortgage effective against third parties?

(a) No, because the effectiveness of this mortgage’s inscription lapsed on March 1, 2013.
(b) No, because the effectiveness of this mortgage’s inscription lapsed on August 1, 2013.
(c) Yes, because this mortgage’s inscription is effective until March 1, 2019.
(d) Yes, because this mortgage’s inscription is effective until August 1, 2019.

A

(A) No, because the effectiveness of this mortgage’s inscription lapsed on March 1, 2013.

(A) is correct because the default ten-year period for duration of inscription applies here. That period is measured from the date of the mortgage, not from the date of the filing, so (B) is wrong. (C) and (D) are wrong because, though the mortgage mentions that it secures a note that matures more than nine years after the date of the mortgage, the mortgage itself does not describe the maturity date (we know the maturity date from another part of the hypothetical), so it is not clear from the mortgage itself that the exceptional longer duration of inscription should apply. If that exception applied, it would be measured from the date of the note’s maturity, March 1, 2013, plus six more years, so (D) would never be the correct answer, and again, (C) would be correct only if the mortgage itself recited the maturity period of the note.

98
Q

((BARBRI))
On June 1, X Bank loans Joe’s Hardware Store money and takes a security interest in Joe’s existing inventory and all after-acquired inventory. A financing statement is filed. On July 1, Y Bank promises to loan Joe’s $10,000 to purchase hammers and obtains a security agreement from Joe’s granting Y Bank a security interest in the hammers. Y Bank immediately files a financing statement and notifies X Bank of the impending loan and its purchase money interest. The loan is then made to Joe’s, which endorses the Y Bank check over to the hammer distributor in exchange for the hammers. Who has priority in the security interest in the hammers?

(a) X Bank, because it has priority in all after-acquired property.
(b) X Bank, because it filed its financing statement first.
(c) Y Bank, because it has an inventory purchase money security interest in the hammers.
(d) Y Bank, because X Bank did not object to the loan.

A

(C) Y Bank, because it has an inventory purchase money security interest in the hammers.

A purchase money security interest in inventory collateral has priority over a conflicting security interest in the same collateral if: (i) it is perfected at the time the debtor gets possession of the collateral; and (ii) any secured party who has perfected her security interest on the same collateral receives authenticated notification of the purchase money security interest before the debtor receives possession of the inventory, and the notification states that the purchase money party has or expects to take a purchase money security interest in the inventory of the debtor described by kind or type. Y Bank, having taken all these steps, has priority over X Bank’s after-acquired property interest.

99
Q

((BARBRI))
A seller and a buyer entered into negotiations over the telephone. They reached a general understanding that the buyer would buy widgets from the seller. Following their conversation, the seller sent the buyer a contract, already signed by the seller, agreeing to sell 1,000 widgets to the buyer for a total contract price of $10,000. Upon receipt of the contract in the mail, the buyer signed the contract and deposited an envelope containing the contract in the mailbox located in front of the buyer’s office building. Before the seller received the contract, the buyer had a change of heart. He telephoned the seller and said, “Look, I just can’t make a profit on those widgets. I’m not interested in that contract we talked about.” The seller replied, “That’s all right, I understand. Maybe we can do business some other time.” The next day, the signed contract was delivered to the seller’s office. The seller, also having had a change of heart, decided that he wanted to enforce the contract. Is the contract enforceable against the buyer?

(a) Yes, because the acceptance occurred prior to rejection.
(b) Yes, because of the parol evidence rule.
(c) No, because the offer to rescind was accepted and that discharged the original contract.
(d) No, because the rejection by telephone voided the acceptance by mail.

A

(A) Yes, because the acceptance occurred prior to rejection.

The contract is enforceable because the “mailbox rule” applies here. Acceptance by mail creates a contract at the moment of transmission, as long as the acceptance is made in a manner and medium suggested by the offer or by another reasonable medium. The buyer’s attempt to reject occurred after acceptance took place. Thus, a valid contract was formed and the seller may enforce it. (B) is incorrect because nothing in the parol evidence rule would serve to validate the contract. Ostensibly, this choice implies that there is a contract because the parol evidence rule will prevent the buyer from introducing evidence of the oral rescission. However, as discussed below, the rescission is ineffective because there was no meeting of the minds. (C) is
incorrect because there is no “meeting of the minds” concerning the rescission. A contract may be discharged by an express agreement between the parties to rescind; the agreement to rescind is itself a binding contract. Because the seller did not know that the buyer had accepted the contract, the seller’s statement that “that’s all right” cannot be construed as acceptance of the buyer’s offer to rescind. Therefore, a contract to rescind was not formed. (D) is incorrect because the telephone rejection did not void the acceptance by mail. As discussed above, if the offeree sends an acceptance first, followed by a rejection, the mailbox rule applies; i.e., a contract is created upon dispatch of the acceptance. Because the buyer’s telephone rejection took place after his acceptance by mail, his acceptance was effective and a contract was created when the letter was mailed.

100
Q

((BARBRI))
On July 1, a rancher offered to sell his prize bull to a breeder for $15,000. On July 10, the breeder wrote the rancher as follows: I have decided to take the bull. A check for $15,000 is enclosed. I am leaving for Argentina for six months and will pick up the bull on January 1. I will pay you for its board and care. The breeder’s letter is:

(a) A counteroffer, because it changes the terms of the offer.
(b) A counteroffer, because it was not a definite expression of acceptance.
(c) An acceptance, and the rancher must board the bull but is entitled to the reasonable value of that service.
(d) An acceptance, and the rancher may refuse to board the bull.

A

(D) An acceptance, and the rancher may refuse to board the bull.

The breeder’s letter is an acceptance, and the rancher may refuse to board the bull. Civil Code article 2601 applies because the parties are contracting for the sale of movables. The parties have agreed on thing and price, so they have a contract. With respect to the sale of the bull, they are merchants. As a result, proposed new terms in this deal between merchants are a part of the contract unless they materially alter the offer. Because the new terms delay deliver for six months, they are likely material alterations that do not become part of the contract and may or may not be accepted by the rancher.

101
Q

((BARBRI))
A homeowner contracted with a builder for the remodeling of the homeowner’s bathroom and kitchen at a cost of $10,000. The contract was in writing and specified that the work was to be completed within two months after the date of execution of the contract. Two weeks after entering into the contract with the homeowner, the builder was offered an extremely lucrative job that would take all of his time and effort for several months. The builder told the homeowner that he was not going to perform. The homeowner diligently called many other contractors over a period of several weeks and none of them could offer a price anywhere near as low as $10,000 for the remodeling work that he wanted done. Two months after entering into the contract with the builder, the homeowner sued the builder for specific performance. What is the likely result of the suit?

(a) The court will order specific performance, because the homeowner was ready and able to perform his part of the contract.
(b) The court will order specific performance, because, despite diligent efforts, the homeowner could find no one who could perform the desired services at a competitive price.
(c) The court will not order specific performance, because the doctrine of laches applies.
(d) The court will not order specific performance, because the contract involves a duty to do.

A

(D) The court will not order specific performance, because the contract involves a duty to do.

Specific performance is within the court’s discretion, but courts more frequently order it for duties not to do or duties to give (execute an instrument). When it comes to the performance of affirmative activity — duties to do — Louisiana courts are generally very hesitant to order specific performance. The court will not order specific performance merely because the builder had a much lower price than anyone else. More likely, the court will require the non-breaching party to hire a different contractor and to sue the builder for damages. The builder would be liable for the difference between the new price for remodeling and the original $10,000 price. It should be noted that nothing indicates that the builder was hired for his unique talents; even if he were, specific performance would not be granted because of difficulties of supervision and a reluctance to force one person to work for another. Thus, (A) and (B) are incorrect. (C) is also incorrect. Laches is a common law defense that does not exist in Louisiana.

102
Q
((BARBRI))
On September 1, a law school professor announced to his class that he would pay the tuition of the bar review course of the student’s choice for the student who received the highest grade in his Constitutional Law class.  A week later, one student in the professor’s class told the professor that he was giving his best effort to get the highest grade in class to win the prize and had already purchased every substantive Constitutional Law outline on the market. Shortly thereafter, the dean of the law school learned of the contest and informed the professor that it was against school policy. On October 1, the professor told his class that he was withdrawing his offer.  Which of the following statements about the professor’s October 1 announcement is correct?  

(a) It had no legal effect, because no offer had been made.
(b) It was an effective revocation of the offer to all students who heard it because it was made in the same manner as the offer.
(c) It was an ineffective revocation as to any student who failed to hear it.
(d) It was ineffective as to the student who had begun performance prior to the withdrawal of the offer.

A

(D) It was ineffective as to the student who had begun performance prior to the withdrawal of the offer.

The professor’s announcement was an ineffective revocation as to the student who had begun performance, because an offer that contemplates acceptance by a completed performance becomes irrevocable once performance has begun. Here, the student’s devotion of his study time extensively to the subject of Constitutional Law and purchase of outlines specifically so he would win the prize constitutes beginning performance, thus making the professor’s offer irrevocable. As to the student who began performance, then, the offer is irrevocable for the reasonable time necessary for him to complete the performance. (A) is incorrect, because the offer was valid. It created a reasonable expectation in the offeree that the offeror was willing to enter into a contract on the basis of the offered terms. There was an expression of a promise, undertaking, or commitment to enter into a contract; the terms were certain and definite; and they were communicated to the offeree. (C) is incorrect because a revocation need only be published in the same
manner as the offer; it need not actually reach everyone who knew of the offer. (B) is incorrect because the start of performance made the offer irrevocable for a reasonable time.